The order of an element in a quotient group











up vote
1
down vote

favorite












Suppose $G$ is a finite group, that $H$ is a subgroup of $G$, and that $N$ is a normal subgroup of $G$. Suppose that $|H| = n$ and $|G| = m|N|$, where $m$ and $n$ are coprime.
Consider the quotient group $G/N$ and let $h in H$. Determine the order of the element $hN$ in the group $G/N$.



Attempt:



Then $|G/N|=m$ , $|H|=n$ , $operatorname{gcd}(m,n)=1$



Order of the element $hN$ in $G/N$ is $k$ where $k$ is the smallest positive integer such that $h^k$ belongs to $N$. $(hN)^k = (h^k)(N)=1 $ iff $h^k$ belongs to $N$.



I am not sure how to use $|H|=n$ and $gcd(m,n)=1$ to find the order of $hN$.










share|cite|improve this question
























  • The order of $hN$ divides the order of $h$.
    – Tortoise
    Nov 19 at 10:16















up vote
1
down vote

favorite












Suppose $G$ is a finite group, that $H$ is a subgroup of $G$, and that $N$ is a normal subgroup of $G$. Suppose that $|H| = n$ and $|G| = m|N|$, where $m$ and $n$ are coprime.
Consider the quotient group $G/N$ and let $h in H$. Determine the order of the element $hN$ in the group $G/N$.



Attempt:



Then $|G/N|=m$ , $|H|=n$ , $operatorname{gcd}(m,n)=1$



Order of the element $hN$ in $G/N$ is $k$ where $k$ is the smallest positive integer such that $h^k$ belongs to $N$. $(hN)^k = (h^k)(N)=1 $ iff $h^k$ belongs to $N$.



I am not sure how to use $|H|=n$ and $gcd(m,n)=1$ to find the order of $hN$.










share|cite|improve this question
























  • The order of $hN$ divides the order of $h$.
    – Tortoise
    Nov 19 at 10:16













up vote
1
down vote

favorite









up vote
1
down vote

favorite











Suppose $G$ is a finite group, that $H$ is a subgroup of $G$, and that $N$ is a normal subgroup of $G$. Suppose that $|H| = n$ and $|G| = m|N|$, where $m$ and $n$ are coprime.
Consider the quotient group $G/N$ and let $h in H$. Determine the order of the element $hN$ in the group $G/N$.



Attempt:



Then $|G/N|=m$ , $|H|=n$ , $operatorname{gcd}(m,n)=1$



Order of the element $hN$ in $G/N$ is $k$ where $k$ is the smallest positive integer such that $h^k$ belongs to $N$. $(hN)^k = (h^k)(N)=1 $ iff $h^k$ belongs to $N$.



I am not sure how to use $|H|=n$ and $gcd(m,n)=1$ to find the order of $hN$.










share|cite|improve this question















Suppose $G$ is a finite group, that $H$ is a subgroup of $G$, and that $N$ is a normal subgroup of $G$. Suppose that $|H| = n$ and $|G| = m|N|$, where $m$ and $n$ are coprime.
Consider the quotient group $G/N$ and let $h in H$. Determine the order of the element $hN$ in the group $G/N$.



Attempt:



Then $|G/N|=m$ , $|H|=n$ , $operatorname{gcd}(m,n)=1$



Order of the element $hN$ in $G/N$ is $k$ where $k$ is the smallest positive integer such that $h^k$ belongs to $N$. $(hN)^k = (h^k)(N)=1 $ iff $h^k$ belongs to $N$.



I am not sure how to use $|H|=n$ and $gcd(m,n)=1$ to find the order of $hN$.







abstract-algebra group-theory normal-subgroups quotient-group






share|cite|improve this question















share|cite|improve this question













share|cite|improve this question




share|cite|improve this question








edited Nov 19 at 0:34

























asked Nov 19 at 0:26









J. Dawson

143




143












  • The order of $hN$ divides the order of $h$.
    – Tortoise
    Nov 19 at 10:16


















  • The order of $hN$ divides the order of $h$.
    – Tortoise
    Nov 19 at 10:16
















The order of $hN$ divides the order of $h$.
– Tortoise
Nov 19 at 10:16




The order of $hN$ divides the order of $h$.
– Tortoise
Nov 19 at 10:16










1 Answer
1






active

oldest

votes

















up vote
1
down vote













I got the idea now. $(|H|,|G/N|)=1$ is key to do this problem. Notice that $|hN|||h|$ (check) and also $|h|||H|$ so that $|hN|||H|$ but $|hN||(G/N)$ since $|H|$ and $|G/N|$ are relatively prime so that $|hN|=1$. Thus $hN=N$ and then $Hsubset N.$ This finishes the proof.






share|cite|improve this answer























    Your Answer





    StackExchange.ifUsing("editor", function () {
    return StackExchange.using("mathjaxEditing", function () {
    StackExchange.MarkdownEditor.creationCallbacks.add(function (editor, postfix) {
    StackExchange.mathjaxEditing.prepareWmdForMathJax(editor, postfix, [["$", "$"], ["\\(","\\)"]]);
    });
    });
    }, "mathjax-editing");

    StackExchange.ready(function() {
    var channelOptions = {
    tags: "".split(" "),
    id: "69"
    };
    initTagRenderer("".split(" "), "".split(" "), channelOptions);

    StackExchange.using("externalEditor", function() {
    // Have to fire editor after snippets, if snippets enabled
    if (StackExchange.settings.snippets.snippetsEnabled) {
    StackExchange.using("snippets", function() {
    createEditor();
    });
    }
    else {
    createEditor();
    }
    });

    function createEditor() {
    StackExchange.prepareEditor({
    heartbeatType: 'answer',
    convertImagesToLinks: true,
    noModals: true,
    showLowRepImageUploadWarning: true,
    reputationToPostImages: 10,
    bindNavPrevention: true,
    postfix: "",
    imageUploader: {
    brandingHtml: "Powered by u003ca class="icon-imgur-white" href="https://imgur.com/"u003eu003c/au003e",
    contentPolicyHtml: "User contributions licensed under u003ca href="https://creativecommons.org/licenses/by-sa/3.0/"u003ecc by-sa 3.0 with attribution requiredu003c/au003e u003ca href="https://stackoverflow.com/legal/content-policy"u003e(content policy)u003c/au003e",
    allowUrls: true
    },
    noCode: true, onDemand: true,
    discardSelector: ".discard-answer"
    ,immediatelyShowMarkdownHelp:true
    });


    }
    });














    draft saved

    draft discarded


















    StackExchange.ready(
    function () {
    StackExchange.openid.initPostLogin('.new-post-login', 'https%3a%2f%2fmath.stackexchange.com%2fquestions%2f3004329%2fthe-order-of-an-element-in-a-quotient-group%23new-answer', 'question_page');
    }
    );

    Post as a guest















    Required, but never shown

























    1 Answer
    1






    active

    oldest

    votes








    1 Answer
    1






    active

    oldest

    votes









    active

    oldest

    votes






    active

    oldest

    votes








    up vote
    1
    down vote













    I got the idea now. $(|H|,|G/N|)=1$ is key to do this problem. Notice that $|hN|||h|$ (check) and also $|h|||H|$ so that $|hN|||H|$ but $|hN||(G/N)$ since $|H|$ and $|G/N|$ are relatively prime so that $|hN|=1$. Thus $hN=N$ and then $Hsubset N.$ This finishes the proof.






    share|cite|improve this answer



























      up vote
      1
      down vote













      I got the idea now. $(|H|,|G/N|)=1$ is key to do this problem. Notice that $|hN|||h|$ (check) and also $|h|||H|$ so that $|hN|||H|$ but $|hN||(G/N)$ since $|H|$ and $|G/N|$ are relatively prime so that $|hN|=1$. Thus $hN=N$ and then $Hsubset N.$ This finishes the proof.






      share|cite|improve this answer

























        up vote
        1
        down vote










        up vote
        1
        down vote









        I got the idea now. $(|H|,|G/N|)=1$ is key to do this problem. Notice that $|hN|||h|$ (check) and also $|h|||H|$ so that $|hN|||H|$ but $|hN||(G/N)$ since $|H|$ and $|G/N|$ are relatively prime so that $|hN|=1$. Thus $hN=N$ and then $Hsubset N.$ This finishes the proof.






        share|cite|improve this answer














        I got the idea now. $(|H|,|G/N|)=1$ is key to do this problem. Notice that $|hN|||h|$ (check) and also $|h|||H|$ so that $|hN|||H|$ but $|hN||(G/N)$ since $|H|$ and $|G/N|$ are relatively prime so that $|hN|=1$. Thus $hN=N$ and then $Hsubset N.$ This finishes the proof.







        share|cite|improve this answer














        share|cite|improve this answer



        share|cite|improve this answer








        edited Nov 19 at 2:13

























        answered Nov 19 at 0:46









        gb2017

        944




        944






























            draft saved

            draft discarded




















































            Thanks for contributing an answer to Mathematics Stack Exchange!


            • Please be sure to answer the question. Provide details and share your research!

            But avoid



            • Asking for help, clarification, or responding to other answers.

            • Making statements based on opinion; back them up with references or personal experience.


            Use MathJax to format equations. MathJax reference.


            To learn more, see our tips on writing great answers.





            Some of your past answers have not been well-received, and you're in danger of being blocked from answering.


            Please pay close attention to the following guidance:


            • Please be sure to answer the question. Provide details and share your research!

            But avoid



            • Asking for help, clarification, or responding to other answers.

            • Making statements based on opinion; back them up with references or personal experience.


            To learn more, see our tips on writing great answers.




            draft saved


            draft discarded














            StackExchange.ready(
            function () {
            StackExchange.openid.initPostLogin('.new-post-login', 'https%3a%2f%2fmath.stackexchange.com%2fquestions%2f3004329%2fthe-order-of-an-element-in-a-quotient-group%23new-answer', 'question_page');
            }
            );

            Post as a guest















            Required, but never shown





















































            Required, but never shown














            Required, but never shown












            Required, but never shown







            Required, but never shown

































            Required, but never shown














            Required, but never shown












            Required, but never shown







            Required, but never shown







            Popular posts from this blog

            Biblatex bibliography style without URLs when DOI exists (in Overleaf with Zotero bibliography)

            ComboBox Display Member on multiple fields

            Is it possible to collect Nectar points via Trainline?